Algorithmic Graph Theory - perfect matching in bipartite graph when det(A) is not 0 The Next CEO of Stack OverflowProving that the characteristic polynomial of a bipartite graph has alternating positive and negative coefficientsLargest Matching whose removal does not leave Eulerian componentsRandomized Algorithm for finding perfect matchingsbipartite graph has perfect matchingHow do you find a perfect bipartite graph using Hall's Theorem?Proving that a Bipartite graph with has a perfect matching$2$-edge-connected graph has perfect matchingMaximum “$2$-to-$1$” matching in a bipartite graphRank of adjacency matrix of twin-free bipartite graph and maximum matchingGraph Theory as an approach to Category Theory

Why did we only see the N-1 starfighters in one film?

Is it safe to use c_str() on a temporary string?

Return the Closest Prime Number

If the heap is initialized for security, then why is the stack uninitialized?

Opposite of a diet

Why doesn't a table tennis ball float on the surface? How do we calculate buoyancy here?

What happens if you roll doubles 3 times then land on "Go to jail?"

Where to find order of arguments for default functions

How should I support this large drywall patch?

Visit to the USA with ESTA approved before trip to Iran

How do spells that require an ability check vs. the caster's spell save DC work?

Can a single photon have an energy density?

Unreliable Magic - Is it worth it?

Which organization defines CJK Unified Ideographs?

Why were Madagascar and New Zealand discovered so late?

Horror movie/show or scene where a horse creature opens its mouth really wide and devours a man in a stables

How to safely derail a train during transit?

Customer Requests (Sometimes) Drive Me Bonkers!

I believe this to be a fraud - hired, then asked to cash check and send cash as Bitcoin

Text adventure game code

What is the purpose of the Evocation wizard's Potent Cantrip feature?

What does this shorthand mean?

How can I open an app using Terminal?

If I blow insulation everywhere in my attic except the door trap, will heat escape through it?



Algorithmic Graph Theory - perfect matching in bipartite graph when det(A) is not 0



The Next CEO of Stack OverflowProving that the characteristic polynomial of a bipartite graph has alternating positive and negative coefficientsLargest Matching whose removal does not leave Eulerian componentsRandomized Algorithm for finding perfect matchingsbipartite graph has perfect matchingHow do you find a perfect bipartite graph using Hall's Theorem?Proving that a Bipartite graph with has a perfect matching$2$-edge-connected graph has perfect matchingMaximum “$2$-to-$1$” matching in a bipartite graphRank of adjacency matrix of twin-free bipartite graph and maximum matchingGraph Theory as an approach to Category Theory










1












$begingroup$


While learning about graphs, I came across theorem that I don't quite understand, and can't find a proof.



If G is bipartite, and $det(A) neq 0,$ then G has a perfect matching. (Given that matrix representation of G is A)



Ps.:
I found bits of information, that proof may be connected with Edmonds Theorem, which I cannot find, since Edmonds-Karp Algorithm is way more popular in google ;)










share|cite|improve this question









New contributor




Yurkee is a new contributor to this site. Take care in asking for clarification, commenting, and answering.
Check out our Code of Conduct.







$endgroup$











  • $begingroup$
    Do you mean bipartite by chance?
    $endgroup$
    – Don Thousand
    yesterday










  • $begingroup$
    Do you mean "bipartite?"
    $endgroup$
    – saulspatz
    yesterday










  • $begingroup$
    thats exactly what I meant, I corrected this typo ;)
    $endgroup$
    – Yurkee
    yesterday










  • $begingroup$
    It's the next-to-last letter we are asking about.
    $endgroup$
    – saulspatz
    yesterday










  • $begingroup$
    Yes, that's the one where vertices can be divided into two disjoint and independent sets.
    $endgroup$
    – Yurkee
    yesterday















1












$begingroup$


While learning about graphs, I came across theorem that I don't quite understand, and can't find a proof.



If G is bipartite, and $det(A) neq 0,$ then G has a perfect matching. (Given that matrix representation of G is A)



Ps.:
I found bits of information, that proof may be connected with Edmonds Theorem, which I cannot find, since Edmonds-Karp Algorithm is way more popular in google ;)










share|cite|improve this question









New contributor




Yurkee is a new contributor to this site. Take care in asking for clarification, commenting, and answering.
Check out our Code of Conduct.







$endgroup$











  • $begingroup$
    Do you mean bipartite by chance?
    $endgroup$
    – Don Thousand
    yesterday










  • $begingroup$
    Do you mean "bipartite?"
    $endgroup$
    – saulspatz
    yesterday










  • $begingroup$
    thats exactly what I meant, I corrected this typo ;)
    $endgroup$
    – Yurkee
    yesterday










  • $begingroup$
    It's the next-to-last letter we are asking about.
    $endgroup$
    – saulspatz
    yesterday










  • $begingroup$
    Yes, that's the one where vertices can be divided into two disjoint and independent sets.
    $endgroup$
    – Yurkee
    yesterday













1












1








1





$begingroup$


While learning about graphs, I came across theorem that I don't quite understand, and can't find a proof.



If G is bipartite, and $det(A) neq 0,$ then G has a perfect matching. (Given that matrix representation of G is A)



Ps.:
I found bits of information, that proof may be connected with Edmonds Theorem, which I cannot find, since Edmonds-Karp Algorithm is way more popular in google ;)










share|cite|improve this question









New contributor




Yurkee is a new contributor to this site. Take care in asking for clarification, commenting, and answering.
Check out our Code of Conduct.







$endgroup$




While learning about graphs, I came across theorem that I don't quite understand, and can't find a proof.



If G is bipartite, and $det(A) neq 0,$ then G has a perfect matching. (Given that matrix representation of G is A)



Ps.:
I found bits of information, that proof may be connected with Edmonds Theorem, which I cannot find, since Edmonds-Karp Algorithm is way more popular in google ;)







graph-theory algebraic-graph-theory bipartite-graph






share|cite|improve this question









New contributor




Yurkee is a new contributor to this site. Take care in asking for clarification, commenting, and answering.
Check out our Code of Conduct.











share|cite|improve this question









New contributor




Yurkee is a new contributor to this site. Take care in asking for clarification, commenting, and answering.
Check out our Code of Conduct.









share|cite|improve this question




share|cite|improve this question








edited yesterday







Yurkee













New contributor




Yurkee is a new contributor to this site. Take care in asking for clarification, commenting, and answering.
Check out our Code of Conduct.









asked yesterday









YurkeeYurkee

1063




1063




New contributor




Yurkee is a new contributor to this site. Take care in asking for clarification, commenting, and answering.
Check out our Code of Conduct.





New contributor





Yurkee is a new contributor to this site. Take care in asking for clarification, commenting, and answering.
Check out our Code of Conduct.






Yurkee is a new contributor to this site. Take care in asking for clarification, commenting, and answering.
Check out our Code of Conduct.











  • $begingroup$
    Do you mean bipartite by chance?
    $endgroup$
    – Don Thousand
    yesterday










  • $begingroup$
    Do you mean "bipartite?"
    $endgroup$
    – saulspatz
    yesterday










  • $begingroup$
    thats exactly what I meant, I corrected this typo ;)
    $endgroup$
    – Yurkee
    yesterday










  • $begingroup$
    It's the next-to-last letter we are asking about.
    $endgroup$
    – saulspatz
    yesterday










  • $begingroup$
    Yes, that's the one where vertices can be divided into two disjoint and independent sets.
    $endgroup$
    – Yurkee
    yesterday
















  • $begingroup$
    Do you mean bipartite by chance?
    $endgroup$
    – Don Thousand
    yesterday










  • $begingroup$
    Do you mean "bipartite?"
    $endgroup$
    – saulspatz
    yesterday










  • $begingroup$
    thats exactly what I meant, I corrected this typo ;)
    $endgroup$
    – Yurkee
    yesterday










  • $begingroup$
    It's the next-to-last letter we are asking about.
    $endgroup$
    – saulspatz
    yesterday










  • $begingroup$
    Yes, that's the one where vertices can be divided into two disjoint and independent sets.
    $endgroup$
    – Yurkee
    yesterday















$begingroup$
Do you mean bipartite by chance?
$endgroup$
– Don Thousand
yesterday




$begingroup$
Do you mean bipartite by chance?
$endgroup$
– Don Thousand
yesterday












$begingroup$
Do you mean "bipartite?"
$endgroup$
– saulspatz
yesterday




$begingroup$
Do you mean "bipartite?"
$endgroup$
– saulspatz
yesterday












$begingroup$
thats exactly what I meant, I corrected this typo ;)
$endgroup$
– Yurkee
yesterday




$begingroup$
thats exactly what I meant, I corrected this typo ;)
$endgroup$
– Yurkee
yesterday












$begingroup$
It's the next-to-last letter we are asking about.
$endgroup$
– saulspatz
yesterday




$begingroup$
It's the next-to-last letter we are asking about.
$endgroup$
– saulspatz
yesterday












$begingroup$
Yes, that's the one where vertices can be divided into two disjoint and independent sets.
$endgroup$
– Yurkee
yesterday




$begingroup$
Yes, that's the one where vertices can be divided into two disjoint and independent sets.
$endgroup$
– Yurkee
yesterday










1 Answer
1






active

oldest

votes


















2












$begingroup$

Let the vertices be $[n]=1,dots,n.$ The theorem follows from the definition of determinant: $$detA=sum_sigmain S_n(-1)^sigma a_isigma(i)$$ where $S_n$ is the set of at permutations on $[n].$ Since $a_ij$ is $0$ or $1$, there must be a permutation $sigma$ such that $i$ and $sigma(i)$ are adjacent for $iin [n].$



Since $sigma$ is a bijection, the two bipartition sets have the same cardinality, and the restriction of $sigma$ to one of them gives a perfect matching.






share|cite|improve this answer









$endgroup$












  • $begingroup$
    Is this proof from the Edmonds Theorem?
    $endgroup$
    – Yurkee
    yesterday






  • 1




    $begingroup$
    Edmonds had a number of theorems. The one I've heard called "Edmonds Theorem" has to do with packing of arborecences, so nothing to do with this.
    $endgroup$
    – saulspatz
    yesterday











Your Answer





StackExchange.ifUsing("editor", function ()
return StackExchange.using("mathjaxEditing", function ()
StackExchange.MarkdownEditor.creationCallbacks.add(function (editor, postfix)
StackExchange.mathjaxEditing.prepareWmdForMathJax(editor, postfix, [["$", "$"], ["\\(","\\)"]]);
);
);
, "mathjax-editing");

StackExchange.ready(function()
var channelOptions =
tags: "".split(" "),
id: "69"
;
initTagRenderer("".split(" "), "".split(" "), channelOptions);

StackExchange.using("externalEditor", function()
// Have to fire editor after snippets, if snippets enabled
if (StackExchange.settings.snippets.snippetsEnabled)
StackExchange.using("snippets", function()
createEditor();
);

else
createEditor();

);

function createEditor()
StackExchange.prepareEditor(
heartbeatType: 'answer',
autoActivateHeartbeat: false,
convertImagesToLinks: true,
noModals: true,
showLowRepImageUploadWarning: true,
reputationToPostImages: 10,
bindNavPrevention: true,
postfix: "",
imageUploader:
brandingHtml: "Powered by u003ca class="icon-imgur-white" href="https://imgur.com/"u003eu003c/au003e",
contentPolicyHtml: "User contributions licensed under u003ca href="https://creativecommons.org/licenses/by-sa/3.0/"u003ecc by-sa 3.0 with attribution requiredu003c/au003e u003ca href="https://stackoverflow.com/legal/content-policy"u003e(content policy)u003c/au003e",
allowUrls: true
,
noCode: true, onDemand: true,
discardSelector: ".discard-answer"
,immediatelyShowMarkdownHelp:true
);



);






Yurkee is a new contributor. Be nice, and check out our Code of Conduct.









draft saved

draft discarded


















StackExchange.ready(
function ()
StackExchange.openid.initPostLogin('.new-post-login', 'https%3a%2f%2fmath.stackexchange.com%2fquestions%2f3164386%2falgorithmic-graph-theory-perfect-matching-in-bipartite-graph-when-deta-is-no%23new-answer', 'question_page');

);

Post as a guest















Required, but never shown

























1 Answer
1






active

oldest

votes








1 Answer
1






active

oldest

votes









active

oldest

votes






active

oldest

votes









2












$begingroup$

Let the vertices be $[n]=1,dots,n.$ The theorem follows from the definition of determinant: $$detA=sum_sigmain S_n(-1)^sigma a_isigma(i)$$ where $S_n$ is the set of at permutations on $[n].$ Since $a_ij$ is $0$ or $1$, there must be a permutation $sigma$ such that $i$ and $sigma(i)$ are adjacent for $iin [n].$



Since $sigma$ is a bijection, the two bipartition sets have the same cardinality, and the restriction of $sigma$ to one of them gives a perfect matching.






share|cite|improve this answer









$endgroup$












  • $begingroup$
    Is this proof from the Edmonds Theorem?
    $endgroup$
    – Yurkee
    yesterday






  • 1




    $begingroup$
    Edmonds had a number of theorems. The one I've heard called "Edmonds Theorem" has to do with packing of arborecences, so nothing to do with this.
    $endgroup$
    – saulspatz
    yesterday















2












$begingroup$

Let the vertices be $[n]=1,dots,n.$ The theorem follows from the definition of determinant: $$detA=sum_sigmain S_n(-1)^sigma a_isigma(i)$$ where $S_n$ is the set of at permutations on $[n].$ Since $a_ij$ is $0$ or $1$, there must be a permutation $sigma$ such that $i$ and $sigma(i)$ are adjacent for $iin [n].$



Since $sigma$ is a bijection, the two bipartition sets have the same cardinality, and the restriction of $sigma$ to one of them gives a perfect matching.






share|cite|improve this answer









$endgroup$












  • $begingroup$
    Is this proof from the Edmonds Theorem?
    $endgroup$
    – Yurkee
    yesterday






  • 1




    $begingroup$
    Edmonds had a number of theorems. The one I've heard called "Edmonds Theorem" has to do with packing of arborecences, so nothing to do with this.
    $endgroup$
    – saulspatz
    yesterday













2












2








2





$begingroup$

Let the vertices be $[n]=1,dots,n.$ The theorem follows from the definition of determinant: $$detA=sum_sigmain S_n(-1)^sigma a_isigma(i)$$ where $S_n$ is the set of at permutations on $[n].$ Since $a_ij$ is $0$ or $1$, there must be a permutation $sigma$ such that $i$ and $sigma(i)$ are adjacent for $iin [n].$



Since $sigma$ is a bijection, the two bipartition sets have the same cardinality, and the restriction of $sigma$ to one of them gives a perfect matching.






share|cite|improve this answer









$endgroup$



Let the vertices be $[n]=1,dots,n.$ The theorem follows from the definition of determinant: $$detA=sum_sigmain S_n(-1)^sigma a_isigma(i)$$ where $S_n$ is the set of at permutations on $[n].$ Since $a_ij$ is $0$ or $1$, there must be a permutation $sigma$ such that $i$ and $sigma(i)$ are adjacent for $iin [n].$



Since $sigma$ is a bijection, the two bipartition sets have the same cardinality, and the restriction of $sigma$ to one of them gives a perfect matching.







share|cite|improve this answer












share|cite|improve this answer



share|cite|improve this answer










answered yesterday









saulspatzsaulspatz

17.1k31435




17.1k31435











  • $begingroup$
    Is this proof from the Edmonds Theorem?
    $endgroup$
    – Yurkee
    yesterday






  • 1




    $begingroup$
    Edmonds had a number of theorems. The one I've heard called "Edmonds Theorem" has to do with packing of arborecences, so nothing to do with this.
    $endgroup$
    – saulspatz
    yesterday
















  • $begingroup$
    Is this proof from the Edmonds Theorem?
    $endgroup$
    – Yurkee
    yesterday






  • 1




    $begingroup$
    Edmonds had a number of theorems. The one I've heard called "Edmonds Theorem" has to do with packing of arborecences, so nothing to do with this.
    $endgroup$
    – saulspatz
    yesterday















$begingroup$
Is this proof from the Edmonds Theorem?
$endgroup$
– Yurkee
yesterday




$begingroup$
Is this proof from the Edmonds Theorem?
$endgroup$
– Yurkee
yesterday




1




1




$begingroup$
Edmonds had a number of theorems. The one I've heard called "Edmonds Theorem" has to do with packing of arborecences, so nothing to do with this.
$endgroup$
– saulspatz
yesterday




$begingroup$
Edmonds had a number of theorems. The one I've heard called "Edmonds Theorem" has to do with packing of arborecences, so nothing to do with this.
$endgroup$
– saulspatz
yesterday










Yurkee is a new contributor. Be nice, and check out our Code of Conduct.









draft saved

draft discarded


















Yurkee is a new contributor. Be nice, and check out our Code of Conduct.












Yurkee is a new contributor. Be nice, and check out our Code of Conduct.











Yurkee is a new contributor. Be nice, and check out our Code of Conduct.














Thanks for contributing an answer to Mathematics Stack Exchange!


  • Please be sure to answer the question. Provide details and share your research!

But avoid


  • Asking for help, clarification, or responding to other answers.

  • Making statements based on opinion; back them up with references or personal experience.

Use MathJax to format equations. MathJax reference.


To learn more, see our tips on writing great answers.




draft saved


draft discarded














StackExchange.ready(
function ()
StackExchange.openid.initPostLogin('.new-post-login', 'https%3a%2f%2fmath.stackexchange.com%2fquestions%2f3164386%2falgorithmic-graph-theory-perfect-matching-in-bipartite-graph-when-deta-is-no%23new-answer', 'question_page');

);

Post as a guest















Required, but never shown





















































Required, but never shown














Required, but never shown












Required, but never shown







Required, but never shown

































Required, but never shown














Required, but never shown












Required, but never shown







Required, but never shown







Popular posts from this blog

Triangular numbers and gcdProving sum of a set is $0 pmod n$ if $n$ is odd, or $fracn2 pmod n$ if $n$ is even?Is greatest common divisor of two numbers really their smallest linear combination?GCD, LCM RelationshipProve a set of nonnegative integers with greatest common divisor 1 and closed under addition has all but finite many nonnegative integers.all pairs of a and b in an equation containing gcdTriangular Numbers Modulo $k$ - Hit All Values?Understanding the Existence and Uniqueness of the GCDGCD and LCM with logical symbolsThe greatest common divisor of two positive integers less than 100 is equal to 3. Their least common multiple is twelve times one of the integers.Suppose that for all integers $x$, $x|a$ and $x|b$ if and only if $x|c$. Then $c = gcd(a,b)$Which is the gcd of 2 numbers which are multiplied and the result is 600000?

Ingelân Ynhâld Etymology | Geografy | Skiednis | Polityk en bestjoer | Ekonomy | Demografy | Kultuer | Klimaat | Sjoch ek | Keppelings om utens | Boarnen, noaten en referinsjes Navigaasjemenuwww.gov.ukOffisjele webside fan it regear fan it Feriene KeninkrykOffisjele webside fan it Britske FerkearsburoNederlânsktalige ynformaasje fan it Britske FerkearsburoOffisjele webside fan English Heritage, de organisaasje dy't him ynset foar it behâld fan it Ingelske kultuergoedYnwennertallen fan alle Britske stêden út 'e folkstelling fan 2011Notes en References, op dizze sideEngland

Հադիս Բովանդակություն Անվանում և նշանակություն | Դասակարգում | Աղբյուրներ | Նավարկման ցանկ